选H时肯定得选K; 若她选择J,则她肯定选择W; 若她选择K,则她肯定选择X。 若该养鱼爱好者选择了X和Z,则下面哪一项是她可能选择的鱼群?

admin2014-11-05  18

问题         选H时肯定得选K;
    若她选择J,则她肯定选择W;
    若她选择K,则她肯定选择X。
若该养鱼爱好者选择了X和Z,则下面哪一项是她可能选择的鱼群?

选项 A、G,H,K
B、G,J,K
C、G,K,L
D、H,J,L

答案C

解析 因为该养鱼爱好者的两种水草已经选好,而由条件(3)的逆否命题可知,J不被选;再由条件(1)可知G和H不能同时被选,因此利用排除法可知C项为正确答案。
转载请注明原文地址:https://kaotiyun.com/show/il4i777K
0

相关试题推荐
最新回复(0)